Author Topic: Problem 2  (Read 23144 times)

Chiara Moraglia

  • Jr. Member
  • **
  • Posts: 9
  • Karma: 1
    • View Profile
Re: Problem 2
« Reply #15 on: September 29, 2012, 09:49:31 PM »
Sorry, I am still having trouble understanding how #2. b is asking to solve for v using v. Could someone help please?

Victor Ivrii

  • Administrator
  • Elder Member
  • *****
  • Posts: 2607
  • Karma: 0
    • View Profile
    • Personal website of Victor Ivrii
Re: Problem 2
« Reply #16 on: September 29, 2012, 11:02:40 PM »
Sorry, I am still having trouble understanding how #2. b is asking to solve for v using v. Could someone help please?

We are looking for $u$, not $v$ -- but $v$ satisfies 1D wave equation and we know everything (well, almost everything) about it
« Last Edit: September 29, 2012, 11:49:46 PM by Victor Ivrii »

Peishan Wang

  • Full Member
  • ***
  • Posts: 32
  • Karma: 6
    • View Profile
Re: Problem 2
« Reply #17 on: September 29, 2012, 11:37:51 PM »
That means in part (c) we don't need to assume that u is even and we will use this assumption in part (d)? Thanks!

Victor Ivrii

  • Administrator
  • Elder Member
  • *****
  • Posts: 2607
  • Karma: 0
    • View Profile
    • Personal website of Victor Ivrii
Re: Problem 2
« Reply #18 on: September 29, 2012, 11:53:34 PM »
You are solving problem from home assignment 2, not from Strauss book. While result will be the same I see no compelling reason to assume a'priory that initial data must be even or odd.

Qitan Cui

  • Jr. Member
  • **
  • Posts: 13
  • Karma: 1
    • View Profile
Re: Problem 2
« Reply #19 on: September 30, 2012, 01:44:14 AM »
Could you give some hints to part (d)? I have r in the denominator and I was thinking if I can make the numerator equal 0 when r=0, so using L'hopital rule the limit might exist. Am I on the right track? Thank you.

Victor Ivrii

  • Administrator
  • Elder Member
  • *****
  • Posts: 2607
  • Karma: 0
    • View Profile
    • Personal website of Victor Ivrii
Re: Problem 2
« Reply #20 on: September 30, 2012, 03:33:53 AM »
I suspect I gave too many hints and this discussion should be stopped.